How can I represent the rotation of a point in $mathbb{R}^2$ graphically?












0












$begingroup$


Let $P = (x, y)$ be a point in the plane; now if I want to rotate it by an angle $theta$ the point $P$ will be moved from $(x, y)$ to $(X,Y)$ where:



$$X = x(cos theta) - y(sin theta)$$
$$Y = x(sin theta) + y(cos theta)$$



Why is this true, how can I represent it graphically?
Can somebody explain me how to get the representative matrix of the rotation function?



Linear transformation of R2: rotation about the origin










share|cite|improve this question











$endgroup$












  • $begingroup$
    Sure. Draw a point on a paper and rotate it some angle. Then try to write the new point in terms of the old. You should be able to get it.
    $endgroup$
    – Dog_69
    Dec 1 '18 at 21:08












  • $begingroup$
    @Dog_69 I don't find intuitive at all what you said there, sorry.
    $endgroup$
    – Ryk
    Dec 1 '18 at 21:24
















0












$begingroup$


Let $P = (x, y)$ be a point in the plane; now if I want to rotate it by an angle $theta$ the point $P$ will be moved from $(x, y)$ to $(X,Y)$ where:



$$X = x(cos theta) - y(sin theta)$$
$$Y = x(sin theta) + y(cos theta)$$



Why is this true, how can I represent it graphically?
Can somebody explain me how to get the representative matrix of the rotation function?



Linear transformation of R2: rotation about the origin










share|cite|improve this question











$endgroup$












  • $begingroup$
    Sure. Draw a point on a paper and rotate it some angle. Then try to write the new point in terms of the old. You should be able to get it.
    $endgroup$
    – Dog_69
    Dec 1 '18 at 21:08












  • $begingroup$
    @Dog_69 I don't find intuitive at all what you said there, sorry.
    $endgroup$
    – Ryk
    Dec 1 '18 at 21:24














0












0








0





$begingroup$


Let $P = (x, y)$ be a point in the plane; now if I want to rotate it by an angle $theta$ the point $P$ will be moved from $(x, y)$ to $(X,Y)$ where:



$$X = x(cos theta) - y(sin theta)$$
$$Y = x(sin theta) + y(cos theta)$$



Why is this true, how can I represent it graphically?
Can somebody explain me how to get the representative matrix of the rotation function?



Linear transformation of R2: rotation about the origin










share|cite|improve this question











$endgroup$




Let $P = (x, y)$ be a point in the plane; now if I want to rotate it by an angle $theta$ the point $P$ will be moved from $(x, y)$ to $(X,Y)$ where:



$$X = x(cos theta) - y(sin theta)$$
$$Y = x(sin theta) + y(cos theta)$$



Why is this true, how can I represent it graphically?
Can somebody explain me how to get the representative matrix of the rotation function?



Linear transformation of R2: rotation about the origin







linear-algebra rotations






share|cite|improve this question















share|cite|improve this question













share|cite|improve this question




share|cite|improve this question








edited Dec 1 '18 at 21:15









Hans Hüttel

3,1972921




3,1972921










asked Dec 1 '18 at 20:29









RykRyk

234




234












  • $begingroup$
    Sure. Draw a point on a paper and rotate it some angle. Then try to write the new point in terms of the old. You should be able to get it.
    $endgroup$
    – Dog_69
    Dec 1 '18 at 21:08












  • $begingroup$
    @Dog_69 I don't find intuitive at all what you said there, sorry.
    $endgroup$
    – Ryk
    Dec 1 '18 at 21:24


















  • $begingroup$
    Sure. Draw a point on a paper and rotate it some angle. Then try to write the new point in terms of the old. You should be able to get it.
    $endgroup$
    – Dog_69
    Dec 1 '18 at 21:08












  • $begingroup$
    @Dog_69 I don't find intuitive at all what you said there, sorry.
    $endgroup$
    – Ryk
    Dec 1 '18 at 21:24
















$begingroup$
Sure. Draw a point on a paper and rotate it some angle. Then try to write the new point in terms of the old. You should be able to get it.
$endgroup$
– Dog_69
Dec 1 '18 at 21:08






$begingroup$
Sure. Draw a point on a paper and rotate it some angle. Then try to write the new point in terms of the old. You should be able to get it.
$endgroup$
– Dog_69
Dec 1 '18 at 21:08














$begingroup$
@Dog_69 I don't find intuitive at all what you said there, sorry.
$endgroup$
– Ryk
Dec 1 '18 at 21:24




$begingroup$
@Dog_69 I don't find intuitive at all what you said there, sorry.
$endgroup$
– Ryk
Dec 1 '18 at 21:24










2 Answers
2






active

oldest

votes


















0












$begingroup$

For any linear transformation $T: V_1 rightarrow V_2$, where $V_1$ and $V_2$ are vector spaces, its matrix representation consists of the images under $T$ of the basis vectors of $V_2$. Since $V_1 = mathbb{R}^2$, we have that the matrix for a counterclockwise rotation of $theta$ is



$$left[begin{array}{ll} cos theta & - sin theta \
sin theta & cos theta end{array}right]$$






share|cite|improve this answer









$endgroup$













  • $begingroup$
    Well I know that from a booklet but how can you actually reach that matrix, what's the proof behind it?
    $endgroup$
    – Ryk
    Dec 1 '18 at 21:22



















0












$begingroup$

Prove the parallelogram rule: if $(x,y)$ and $(x',y')$ are any two points, then the four points $(0,0)$, $(x,y)$, $(x',y')$ and $(x+x',y+y')$ form a parallelogram.



Now: Where does $(1,0)$ go when rotated by $theta$? Where does $(0,1)$ go? (Use the definitions of sine and cosine.)



Where does $(x,0)$ go when rotated by $theta$? Where does $(0,y)$ go? (Use the results of the last paragraph.)



Where does $(x,y)$ go when rotated by $theta$? (Hint: use the parallelogram rule on the points $(x,0)$ and $(0,y)$. Notice that when you rotate a parallelogram, you get another parallelogram. Also notice that a rectangle is a special type of parallelogram.)






share|cite|improve this answer









$endgroup$













    Your Answer





    StackExchange.ifUsing("editor", function () {
    return StackExchange.using("mathjaxEditing", function () {
    StackExchange.MarkdownEditor.creationCallbacks.add(function (editor, postfix) {
    StackExchange.mathjaxEditing.prepareWmdForMathJax(editor, postfix, [["$", "$"], ["\\(","\\)"]]);
    });
    });
    }, "mathjax-editing");

    StackExchange.ready(function() {
    var channelOptions = {
    tags: "".split(" "),
    id: "69"
    };
    initTagRenderer("".split(" "), "".split(" "), channelOptions);

    StackExchange.using("externalEditor", function() {
    // Have to fire editor after snippets, if snippets enabled
    if (StackExchange.settings.snippets.snippetsEnabled) {
    StackExchange.using("snippets", function() {
    createEditor();
    });
    }
    else {
    createEditor();
    }
    });

    function createEditor() {
    StackExchange.prepareEditor({
    heartbeatType: 'answer',
    autoActivateHeartbeat: false,
    convertImagesToLinks: true,
    noModals: true,
    showLowRepImageUploadWarning: true,
    reputationToPostImages: 10,
    bindNavPrevention: true,
    postfix: "",
    imageUploader: {
    brandingHtml: "Powered by u003ca class="icon-imgur-white" href="https://imgur.com/"u003eu003c/au003e",
    contentPolicyHtml: "User contributions licensed under u003ca href="https://creativecommons.org/licenses/by-sa/3.0/"u003ecc by-sa 3.0 with attribution requiredu003c/au003e u003ca href="https://stackoverflow.com/legal/content-policy"u003e(content policy)u003c/au003e",
    allowUrls: true
    },
    noCode: true, onDemand: true,
    discardSelector: ".discard-answer"
    ,immediatelyShowMarkdownHelp:true
    });


    }
    });














    draft saved

    draft discarded


















    StackExchange.ready(
    function () {
    StackExchange.openid.initPostLogin('.new-post-login', 'https%3a%2f%2fmath.stackexchange.com%2fquestions%2f3021792%2fhow-can-i-represent-the-rotation-of-a-point-in-mathbbr2-graphically%23new-answer', 'question_page');
    }
    );

    Post as a guest















    Required, but never shown

























    2 Answers
    2






    active

    oldest

    votes








    2 Answers
    2






    active

    oldest

    votes









    active

    oldest

    votes






    active

    oldest

    votes









    0












    $begingroup$

    For any linear transformation $T: V_1 rightarrow V_2$, where $V_1$ and $V_2$ are vector spaces, its matrix representation consists of the images under $T$ of the basis vectors of $V_2$. Since $V_1 = mathbb{R}^2$, we have that the matrix for a counterclockwise rotation of $theta$ is



    $$left[begin{array}{ll} cos theta & - sin theta \
    sin theta & cos theta end{array}right]$$






    share|cite|improve this answer









    $endgroup$













    • $begingroup$
      Well I know that from a booklet but how can you actually reach that matrix, what's the proof behind it?
      $endgroup$
      – Ryk
      Dec 1 '18 at 21:22
















    0












    $begingroup$

    For any linear transformation $T: V_1 rightarrow V_2$, where $V_1$ and $V_2$ are vector spaces, its matrix representation consists of the images under $T$ of the basis vectors of $V_2$. Since $V_1 = mathbb{R}^2$, we have that the matrix for a counterclockwise rotation of $theta$ is



    $$left[begin{array}{ll} cos theta & - sin theta \
    sin theta & cos theta end{array}right]$$






    share|cite|improve this answer









    $endgroup$













    • $begingroup$
      Well I know that from a booklet but how can you actually reach that matrix, what's the proof behind it?
      $endgroup$
      – Ryk
      Dec 1 '18 at 21:22














    0












    0








    0





    $begingroup$

    For any linear transformation $T: V_1 rightarrow V_2$, where $V_1$ and $V_2$ are vector spaces, its matrix representation consists of the images under $T$ of the basis vectors of $V_2$. Since $V_1 = mathbb{R}^2$, we have that the matrix for a counterclockwise rotation of $theta$ is



    $$left[begin{array}{ll} cos theta & - sin theta \
    sin theta & cos theta end{array}right]$$






    share|cite|improve this answer









    $endgroup$



    For any linear transformation $T: V_1 rightarrow V_2$, where $V_1$ and $V_2$ are vector spaces, its matrix representation consists of the images under $T$ of the basis vectors of $V_2$. Since $V_1 = mathbb{R}^2$, we have that the matrix for a counterclockwise rotation of $theta$ is



    $$left[begin{array}{ll} cos theta & - sin theta \
    sin theta & cos theta end{array}right]$$







    share|cite|improve this answer












    share|cite|improve this answer



    share|cite|improve this answer










    answered Dec 1 '18 at 21:20









    Hans HüttelHans Hüttel

    3,1972921




    3,1972921












    • $begingroup$
      Well I know that from a booklet but how can you actually reach that matrix, what's the proof behind it?
      $endgroup$
      – Ryk
      Dec 1 '18 at 21:22


















    • $begingroup$
      Well I know that from a booklet but how can you actually reach that matrix, what's the proof behind it?
      $endgroup$
      – Ryk
      Dec 1 '18 at 21:22
















    $begingroup$
    Well I know that from a booklet but how can you actually reach that matrix, what's the proof behind it?
    $endgroup$
    – Ryk
    Dec 1 '18 at 21:22




    $begingroup$
    Well I know that from a booklet but how can you actually reach that matrix, what's the proof behind it?
    $endgroup$
    – Ryk
    Dec 1 '18 at 21:22











    0












    $begingroup$

    Prove the parallelogram rule: if $(x,y)$ and $(x',y')$ are any two points, then the four points $(0,0)$, $(x,y)$, $(x',y')$ and $(x+x',y+y')$ form a parallelogram.



    Now: Where does $(1,0)$ go when rotated by $theta$? Where does $(0,1)$ go? (Use the definitions of sine and cosine.)



    Where does $(x,0)$ go when rotated by $theta$? Where does $(0,y)$ go? (Use the results of the last paragraph.)



    Where does $(x,y)$ go when rotated by $theta$? (Hint: use the parallelogram rule on the points $(x,0)$ and $(0,y)$. Notice that when you rotate a parallelogram, you get another parallelogram. Also notice that a rectangle is a special type of parallelogram.)






    share|cite|improve this answer









    $endgroup$


















      0












      $begingroup$

      Prove the parallelogram rule: if $(x,y)$ and $(x',y')$ are any two points, then the four points $(0,0)$, $(x,y)$, $(x',y')$ and $(x+x',y+y')$ form a parallelogram.



      Now: Where does $(1,0)$ go when rotated by $theta$? Where does $(0,1)$ go? (Use the definitions of sine and cosine.)



      Where does $(x,0)$ go when rotated by $theta$? Where does $(0,y)$ go? (Use the results of the last paragraph.)



      Where does $(x,y)$ go when rotated by $theta$? (Hint: use the parallelogram rule on the points $(x,0)$ and $(0,y)$. Notice that when you rotate a parallelogram, you get another parallelogram. Also notice that a rectangle is a special type of parallelogram.)






      share|cite|improve this answer









      $endgroup$
















        0












        0








        0





        $begingroup$

        Prove the parallelogram rule: if $(x,y)$ and $(x',y')$ are any two points, then the four points $(0,0)$, $(x,y)$, $(x',y')$ and $(x+x',y+y')$ form a parallelogram.



        Now: Where does $(1,0)$ go when rotated by $theta$? Where does $(0,1)$ go? (Use the definitions of sine and cosine.)



        Where does $(x,0)$ go when rotated by $theta$? Where does $(0,y)$ go? (Use the results of the last paragraph.)



        Where does $(x,y)$ go when rotated by $theta$? (Hint: use the parallelogram rule on the points $(x,0)$ and $(0,y)$. Notice that when you rotate a parallelogram, you get another parallelogram. Also notice that a rectangle is a special type of parallelogram.)






        share|cite|improve this answer









        $endgroup$



        Prove the parallelogram rule: if $(x,y)$ and $(x',y')$ are any two points, then the four points $(0,0)$, $(x,y)$, $(x',y')$ and $(x+x',y+y')$ form a parallelogram.



        Now: Where does $(1,0)$ go when rotated by $theta$? Where does $(0,1)$ go? (Use the definitions of sine and cosine.)



        Where does $(x,0)$ go when rotated by $theta$? Where does $(0,y)$ go? (Use the results of the last paragraph.)



        Where does $(x,y)$ go when rotated by $theta$? (Hint: use the parallelogram rule on the points $(x,0)$ and $(0,y)$. Notice that when you rotate a parallelogram, you get another parallelogram. Also notice that a rectangle is a special type of parallelogram.)







        share|cite|improve this answer












        share|cite|improve this answer



        share|cite|improve this answer










        answered Dec 1 '18 at 21:30









        Akiva WeinbergerAkiva Weinberger

        13.8k12167




        13.8k12167






























            draft saved

            draft discarded




















































            Thanks for contributing an answer to Mathematics Stack Exchange!


            • Please be sure to answer the question. Provide details and share your research!

            But avoid



            • Asking for help, clarification, or responding to other answers.

            • Making statements based on opinion; back them up with references or personal experience.


            Use MathJax to format equations. MathJax reference.


            To learn more, see our tips on writing great answers.




            draft saved


            draft discarded














            StackExchange.ready(
            function () {
            StackExchange.openid.initPostLogin('.new-post-login', 'https%3a%2f%2fmath.stackexchange.com%2fquestions%2f3021792%2fhow-can-i-represent-the-rotation-of-a-point-in-mathbbr2-graphically%23new-answer', 'question_page');
            }
            );

            Post as a guest















            Required, but never shown





















































            Required, but never shown














            Required, but never shown












            Required, but never shown







            Required, but never shown

































            Required, but never shown














            Required, but never shown












            Required, but never shown







            Required, but never shown







            Popular posts from this blog

            Le Mesnil-Réaume

            Ida-Boy-Ed-Garten

            web3.py web3.isConnected() returns false always